¡un ejemplo en el que cambiar el marco de referencia de un observador cambia el resultado de los eventos!

Considere dos cargas idénticas que se mueven con velocidad uniforme. Habrá una fuerza magnética de atracción entre ellos, ya que dos corrientes en la misma dirección se atraen entre sí. Si me siento en una de las cargas, según yo, la otra carga no se mueve. Entonces no habrá ninguna atracción magnética. ¿Cómo cambia el marco de referencia el resultado de la interacción? ¿Qué es lo que realmente sucederá?

En primer lugar, existe una fuerza de repulsión electrostática entre las dos cargas del mismo signo,
F = q 1 q 2 4 π ϵ 0 r 2
Eso es todo en el cuadro donde las cargas están en reposo. En un marco que se mueve por la velocidad v perpendicular a la separación de las cargas, ambas cargas se ven γ veces más pesados ​​porque se están moviendo. Por lo tanto, la misma fuerza electrostática causará una aceleración más baja, lo cual está bien; puede interpretarse como una dilatación del tiempo. También habrá un campo magnético. v × mi / C 2 en ese marco en movimiento.
Este campo magnético adicional B de hecho, contribuirá con una atracción positiva que puede calcular: reducirá aún más la fuerza de repulsión original. Si combina todas estas cosas, verá que las cargas se aceleran como se esperaba debido a una transformación de las trayectorias.
Visite la página: Electromagnetismo clásico y Relatividad especial . Lea hasta que encuentre las palabras "Esto no significa que se ven dos conjuntos de eventos completamente diferentes en los dos marcos, sino que la misma secuencia de eventos se describe de dos maneras diferentes", su pregunta será respondida.
Enlaces útiles: publicación en física SE que contiene dos buenas respuestas, relacionadas con su pregunta: physics.stackexchange.com/questions/38151/… Y un enlace al capítulo 13-6 del Vol II de Feynman, que trata la relatividad de los campos magnéticos (hecho libre- para leer en línea por Caltech) feynmanlectures.caltech.edu/II_13.html#Ch13-S6

Respuestas (2)

Tienes razón; en el marco que se mueve con las cargas no habrá atracción magnética. Sin embargo, habrá una repulsión electrostática, por lo que verá que las dos partículas se alejan una de la otra.

Considere otro marco (el resto del marco) donde ambas partículas se mueven una al lado de la otra en la dirección x (supongo que esto es lo que está imaginando). Las partículas aún se repelen electrostáticamente, ¡pero ahora también se atraen magnéticamente!

Esto puede parecer un problema, pero no lo es porque aunque la fuerza medida en el marco de reposo es más débil ya que la repulsión electrostática es parcialmente cancelada por la atracción magnética, el tiempo experimentado por los observadores en movimiento también es diferente debido a la brujería de relatividad especial. Si el marco que se mueve con las cargas está cebado y el resto del marco no está cebado, entonces

t = γ t F = q ( mi + v × B ) = q ( mi v B ) F = q mi

Lo que buscamos mostrar es que dado que la fuerza es inversamente proporcional al cuadrado del tiempo, F y F' están relacionados por un factor de γ 2 en cuyo caso, los observadores en cualquiera de los marcos calcularán las trayectorias correspondientes utilizando las ecuaciones de Maxwell.

Ya que para una carga puntual

mi = 1 4 π ϵ q r 2

B = m 4 π q v × r ^ r 2 = m 4 π q v r 2

Ahora tenemos todo lo que necesitamos. Conectando B a F y usando m = 1 / ( C 2 ϵ ) y γ = 1 / 1 v 2 C 2

F = q ( mi v B ) = q ( mi v 2 C 2 1 4 π ϵ q r 2 ) = q γ 2 mi

F = q mi = q mi = 1 γ 2 F

Que es lo que queríamos todo el tiempo desde

F = d 2 y d t 2 = d 2 y ( γ   d t ) 2 = 1 γ 2 F

(teniendo en cuenta que dy no se ve afectado por un impulso en la dirección x)

Para resolver problemas más complicados de este tipo, generalmente desea hablar sobre tensores de Faraday, 4 corrientes y notación de Einstein, pero espero que este ejemplo simple le haya dado una idea de cómo se relacionan el electromagnetismo y la relatividad especial. Es posible que te hayas preguntado qué pasa si las partículas van tan rápido que la fuerza magnética supera la repulsión electrostática, ¡pero de hecho puedes ver que para hacerlo debes superar la velocidad de la luz!

Si desea comprender por qué el electromagnetismo es manifiestamente robusto frente a las paradojas de la relatividad, querrá aprender sobre el análisis de tensores en el espacio-tiempo 4D, que es un conocimiento imprescindible para cualquier físico que se precie y se puede encontrar en la parte posterior de la mayoría de las mecánicas o libros de texto de electrodinámica o al comienzo de la mayoría de los libros de relatividad general. Si desea obtener cierta intuición física sobre cómo funciona EM, le recomiendo Purcell simplificado.

http://física.weber.edu/schroeder/mrr/MRRtalk.html

En las conferencias de Feynamn, muestra de una manera bastante directa, tomando el ejemplo de una carga que se mueve paralelamente a un cable, que una descripción electromagnética completa es invariable al marco de referencia inercial, es decir, la electricidad y el magnetismo tomados en conjunto son consistentes con la teoría de Einstein . relatividad.

Por lo que en casos, como el de tu ejemplo, siempre debes tener en cuenta cómo se mezcla la electricidad mi y magnético B cambios de campo de un observador a otro. Algunas citas de su libro:

Resulta que cualquier marco inercial servirá. También veremos que el magnetismo y la electricidad no son cosas independientes, que siempre deben tomarse juntos como un campo electromagnético completo. Aunque en el caso estático las ecuaciones de Maxwell se separan en dos pares distintos, uno para la electricidad y otro para el magnetismo, sin conexión aparente entre los dos campos, sin embargo, en la naturaleza misma existe una relación muy íntima entre ellos que surge del principio de la relatividad Históricamente, el principio de la relatividad se descubrió después de las ecuaciones de Maxwell. De hecho, fue el estudio de la electricidad y el magnetismo lo que condujo finalmente al descubrimiento de Einstein de su principio de relatividad.

Un ejemplo de los factores relativistas que entran en juego, utilizando las leyes del electromagnetismo, antes de la relatividad especial, se puede encontrar aquí , si está interesado.

Más adelante en su derivación, verá que las densidades de carga también dependen del marco, para que se cumpla la ley de conservación de la carga total. ¡Limpio!

Para cortar al resultado final:

Hemos encontrado que obtenemos el mismo resultado físico si analizamos el movimiento de una partícula que se mueve a lo largo de un alambre en un sistema de coordenadas en reposo con respecto al alambre, o en un sistema en reposo con respecto a la partícula. En el primer caso, la fuerza era puramente "magnética", en el segundo, era puramente "eléctrica".

Comencé con el ejemplo de Feynman, solo para establecer un poco de contexto, ahora volvamos a nuestro caso, sin cable, como señaló Luboš en los comentarios, en el marco de referencia de las cargas en movimiento, solo existe la repulsión de Coulomb entre los dos protones (o dos electrones), mientras que en el marco del laboratorio (cargas en movimiento) la repulsión de Coulomb se acopla con una fuerza magnética que atrae a los dos. Siguiendo el razonamiento presentado anteriormente, la transición de campo eléctrico y magnético entre los dos marcos mencionados, se obtiene simplemente aplicando las transformaciones de Lorentz en los campos de un marco para obtener el otro. Aquí, para nuestro problema específico, solo podemos trabajar con los componentes transversales de mi , denotado como mi t .

En el resto del cuadro de cargos, S , solo tenemos un campo eléctrico, su componente transversal denotada mi t , la transformada de Lorentz que da el campo eléctrico transversal en S es:

mi t = γ ( mi t | v × B | )

¡Observe la aparición de un campo magnético en la expresión!

Ahora que hemos definido los campos en ambos marcos, sabemos que las cargas estarán sujetas a fuerzas EM en cada marco, así que para verificar la física, averigüemos cuáles son los momentos transversales de cada carga, después de que la fuerza EM haya actuado. en ellos por un tiempo determinado. Esperamos que los resultados sean los mismos en ambos marcos.

Empleando la ecuación de movimiento relativista correcta F = d pag / d t , después del intervalo de tiempo Δ t esperamos que el cambio de cantidad de movimiento transversal sea Δ pag t = F Δ t en S , y Δ pag t = F Δ t en S . Obtener el mismo resultado del cambio de momento transversal en ambos marcos, significa que se debe cumplir la siguiente relación:

Δ pag t Δ pag t = 1 = F Δ t F Δ t

Donde para cualquier intervalo de tiempo elegido Δ t en S , el intervalo de tiempo correspondiente en S está dilatado (dilatación del tiempo en marcos móviles):

Δ t = γ Δ t

Ahora solo falta encontrar la expresión de F , el resultado esperado debe cancelar el efecto de la dilatación del tiempo, para que se conserve el impulso en la dirección transversal. Sabemos que en S cada carga también está sujeta a un campo magnético, y su efecto debería reducir la repulsión de Coulomb. Conocimiento mi t (de antes), tenemos la fuerza EM total sobre cada carga en S es dado por:

F = γ q ( mi t v B ) = γ q ( mi t v ( v mi t C 2 ) ) F = γ q mi t ( 1 v 2 C 2 )

Sustituyendo Δ t y F t de las expresiones anteriores se obtiene:

F Δ t F Δ t = F Δ t γ 2 ( 1 v 2 C 2 ) F Δ t = 1
Que de hecho es igual a 1, como γ = ( 1 v 2 / C 2 ) 1 / 2 .

Por lo tanto, se obtiene el mismo resultado físico de cualquier marco de referencia, S y S , ninguna paradoja! Para concluir, una cita de Feynman nuevamente:

Una descripción electromagnética completa es invariable; la electricidad y el magnetismo en conjunto son consistentes con la relatividad de Einstein.

Otras lecturas útiles: conferencias de Feynman, Vol II capítulo 13-6.

Transformación de Lorentz de los campos

Transformación de Lorentz para EM, de wikipedia.